Select ALL the functions whose graphs include the point (16, 4).

Select ALL The Functions Whose Graphs Include The Point (16, 4).

Answers

Answer 1

Answer:

D

Step-by-step explanation:

x=16

y=4

4=16-12

4=4

Answer 2

Answer:

D

Step-by-step explanation:


Related Questions

URGENT HELP NEEDED!!!!!

Find the simple interest earned at the end of 1 year. Show your calculations as best as possible (1 mark) correct answer (1mark) P = $1800 R = 5% T = 1 year


What is the total amount earned at the end of 3 years? P = $2600 R = 12% T = 3 years
1 point
$936
$3 536
$93 600
Both A and B


What principal would give an interest of $36 in 3 years at 3% p.a.? Show your calculations as best as possible (1 mark) correct answer (1mark)


In how many years will $600 double itself at 10% simple interest?
1 point
8 years and 4 months
8 years and 1/3
100 Months
All of the above
Both A and B


In what time will $400 amount to $512 if the simple interest is the calculated at 14% p.a.? Write your answer


Imagine you have a friend who doesn't quite understand what investing money into a bank means. Think of a financial situation and use the following words and numbers to help teach them. (principal $1200, interest earned, interest rate 4%, total amount, term, per anum). This is worth 8 marks.

Answers

Answer:

k

Step-by-step explanation:

The Titanus beetle can measure 16710 centimeters in length, and the Giant Weta beetle can measure 8510 centimeters in length. How much longer is the Titanus than the Giant Weta?

Answers

Answer:

The answer you're looking for is 8,200cm

Step-by-step explanation:

(Trigonometry) The ladder leaning against a building makes an angle of 78° with the ground. The foot of the ladder is 5 feet from the building. How many feet long is the ladder? Round your answer to the nearest whole number. (Hint: DRAW A PICTURE)

Answers

The foot of the ladder is
10
feet away from the base of the wall. [Ans]

find the product (3n-4)(3n-4)

Answers

Answer:

9n-8 is the product

Answer:

Search Quickmath in Chrome and

all problem will solve... I hope it helps and trust me it works

Pls help me I don’t understand this!!!!

Answers

Answer: D

Step-by-step explanation:

D The angle agacent to arc ML is half the degree. Your answer is 118

Answer:

D

Step-by-step explanation:

angle J is a circumference angle that insists on arc ML. the measure the angle  of this arc is double of it

59 * 2 = 118 degrees

what is 3x+8 + 4x+10

Answers

7x+18-answer

Step-by-step explanation:

3

+

8

+

4

+

1

0

3x+8+4x+10

3x+8+4x+10

Simplify

1

Add the numbers

3

+

8

+

4

+

1

0

3x+{\color{#c92786}{8}}+4x+{\color{#c92786}{10}}

3x+8+4x+10

3

+

1

8

+

4

3x+{\color{#c92786}{18}}+4x

3x+18+4x

2

Combine like terms

Answer:

[tex]\huge\boxed{Answer\hookleftarrow}[/tex]

[tex]3x + 8 + 4x + 10[/tex]

Bring the like terms together & add them.

[tex]3x + 8 + 4x + 10 \\ = 3x + 4x + 8 + 10 \\ = 7x + 18[/tex]

[tex]\underline{\textsf{\textbf{Answer = 7x + 18}}}[/tex]

____________________

ʰᵒᵖᵉ ⁱᵗ ʰᵉˡᵖˢ

╭═══════ღ❦ღ══╮

[tex]RainbowSalt2^{2}2^{2}[/tex]

╰══ღ❦ღ═══════╯

(PLEASE HELP 30 POINTS)
Select all the correct answers.
Liam owns some rectangular plots of land. All of the plots are the same length, x, and the width of each plot is 5 yards less than the length. The
total number of plots Liam owns is 20 more than the length of a plot. If the total area of all the plots Liam owns is 2,688 square yards, which
statements about the length of each plot are true?
The equation x3 - 15x2 - 100x - 2,688 0 can be used to find the length of each plot.
The equation x3 + 25x2 + 100% -2,688 = 0 can be used to find the length of each plot.
o o o o o
The equation x3 + 15x2 - 100x - 2,688 = 0 can be used to find the length of each plot.
The length of each plot is 12 yards.
The length of each plot is 8 yards.

Answers

Answer:

We have to:

"All of the plots are the same length, x"

L = x

"and the width of each plot is 5 yards less than the length"

W = x-5

"The total number of plots Liam owns is 20 more than the length of a plot"

20 + x

"the total area of all the plots Liam owns is 2,688 square yards"

A = (20 + x) * (x) * (x-5)

A = (20x - 100 + x ^ 2 -5x) * (x)

A = (x ^ 2 + 15x - 100) * (x)

2688 = (x ^ 3 + 15x ^ 2 - 100x)

x ^ 3 + 15x ^ 2 - 100x = 2688

x ^ 3 + 15x ^ 2 - 100x - 2688 = 0

Answer:

*** The equation x3 + 15x2 - 100x - 2.688 = 0 can be used to find the length of each plot.

Answer:x^3+15x^2-100x-2,688=0

Step-by-step explanation:

Which equation can be used to solve a - 5 = 30?A. a = 30 - 5 B. a = 30 + 5 C a = 30 + 5 = 5 D a = 30.5​

Answers

Which equation can be used to solve a - 5 = 30

answer= a=30+5

HOPE THIS HELPS YOU......

Today, the population of Canyon Falls is 22{,}50022,50022, comma, 500 and the population of Swift Creek is 15{,}20015,20015, comma, 200. The population of Canyon Falls is decreasing at the rate of 740740740 people each year while the population of Swift Creek is increasing at the rate of 1{,}5001,5001, comma, 500 people each year. Assuming these rates continue into the future, in how many years from today will the population of Swift Creek equal twice the population of Canyon Falls

Answers

Answer:

10 years

Step-by-step explanation:

Given :

Population of Canyon falls = 22500

Rate of decrease = 740 per year

Population after x years :

f(x) = 22500 - 740x ; x = number of years

Population of Swift Creek = 15200

Rate of Increase = 1500 per year

Population after x years :

f(x) = 15200 + 1500x ; x = number of years

Number of years when, population of swift creek will be twice that of Canyon falls

15200 + 1500x = 2(22500 - 740x)

15200 + 1500x = 45000 - 1480x

15200 - 45000 = - 1480x - 1500x

-29800 = - 2980x

x = 29800 / 2980

x = 10

After 10 years

A hiker is standing 40 feet away from a tree that is 50 feet tall.

What is the angle of elevation from the hikers foot to the top of the tree??


HELP ME PLEASE ITS URGENT

Answers

Answer:

87.9°

Step-by-step explanation:

arctan(50/40)=87.9°

A rectangle has an area of 4x²-8x+3 square units. Which of the following could represent the perimeter of the rectangle in terms of x?
a. 2x-1
b. 2x-3
c. 4x-4
d. 8x-8
e. 10x-8

Answers

I believe the answer is B

At what rate, in minutes per lap, does this swimmer swim?

Answers

Answer:

1.6

Step-by-step explanation:

all u gotta do is divide 6.4 by 4:)

hope that helps!<3

What is the original price of an item cost if the tax is 9.25% and the discount is 25% and the new price after discount and tax is $62.28?

Answers

Answer: $ 91.50.

Step-by-step explanation:

Let x be the original price.

Since discount is applied before tax.

New price = (Original price - Discount)-Tax rate (Original price - Discount)

, where Discount = Discount rate x Original price.

Substituting values, we get

[tex]62.28=(x-0.25x)-0.0925(x-0.25x)[/tex]

[tex]62.28=(0.75x)-0.0925(0.75x)[/tex]

[tex]62.28=0.75x-0.069375x[/tex]

[tex]62.28=0.680625x[/tex]

[tex]x=\frac{62.28}{0.680625}[/tex]

[tex]x=91.50[/tex]

Hence, the original price was  $ 91.50.

i will give brainliest.

Answers

Answer:

speed

Step-by-step explanation:

I need help please and thank you!!

Answers

Answer:

Option 4

Step-by-step explanation:

2x² + 32 = 0

2x² = -32

x² = -16

sqrt(-16) got no real solution. (no real number multiplied by itself will be negative)

I think The answer is 4

What is the equation of a circle with center (7, 2) and radius 2?

Answers

Answer:

(x - 7)² + (y - 2)² = 4

Step-by-step explanation:

The equation of a circle in standard form is

(x - h)² + (y - k)² = r²

where (h, k) are the coordinates of the centre and r is the radius

Here (h, k ) = (7, 2 ) and r = 2 , then

(x - 7)² + (y - 2)² = 2² , that is

(x - 7)² + (y - 2)² = 4 ← equation of circle

The area of a rectangle is 80 ft2. If the rectangle is 8 feet long, what is its width?

Answers

the width is 10. l x w = a. 8 x 10 = 80
The width is 10 because the area of a rectangle is lxw so I can do the opposite of multiplication which is division and I then divide 80 by 8 and I get 10

Line JK passes through points J(–3, 11) and K(1, –3). What is the equation of line JK in standard form?

7x + 2y = –1
7x + 2y = 1
14x + 4y = –1
14x + 4y = 1

Answers

Answer:

7x + 2y = 1

Step-by-step explanation:

First we find the slope m of the line

The slope of a line that passes through 2 points x1, y1 and x2 , y2 is given as

m = (y2 - y1)/(x2 - x1)

Considering the points (–3, 11) and K(1, –3), the slope of the line m

= (-3-11)/(1 --3)

= -14/4

= 7/2

the line JK may be given as

y - 11 = -7/2(x - -3)

multiply through the equation by 2

2y - 22 = -7(x + 3)

2y - 22 = -7x - 21

7x + 2y = 22 - 21

7x + 2y = 1

Answer:

B

Step-by-step explanation:

7x + 2y = 1

The actual amount in a 12-ounce container of a certain brand of orange juice is normally distributed with mean μ = 12.34 ounces and standard deviation σ = 0.04 ounce. What percentage of the juice bottles contain between 12.24 and 12.30 ounces of orange juice?

Answers

Answer:

15.25%

Step-by-step explanation:

The actual amount in a 12-ounce container of a certain brand of orange juice is normally distributed with mean μ = 12.34 ounces and standard deviation σ = 0.04 ounce. What percentage of the juice bottles contain between 12.24 and 12.30 ounces of orange juice?

We solve using the z score formula

z-score is is z = (x-μ)/σ,

where x is the raw score

μ is the population mean = μ = 12.34 ounces

σ is the population standard deviation = σ = 0.04 ounce

For x = 12.24

z = 12.24 - 12.34/0.04

z = -2.5

Probability value from Z-Table:

P(x = 12.24) = 0.0062097

For x = 12.30

z= 12.30 - 12.34/0.04

z = -1

Probability value from Z-Table:

P(x = 12.30) = 0.15866

Hence, the probability of the juice bottles contain between 12.24 and 12.30 ounces of orange juice

P(x = 12.30) - P(x = 12.24)

= 0.15866 - 0.0062097

= 0.1524503

Therefore, the percentage of the juice bottles contain between 12.24 and 12.30 ounces of orange juice is calculated as:

= 0.1524503 × 100

= 15.24503%

= 15.25%

a=8 and b=2.25 but what is k

Answers

Answer:

a = 8, b = 2.25, k = 27

Step-by-step explanation:

y = abˣ

Points on the graph:

(0, 8), (1, 18), (1.5, k)

Substitute coordinates into equation and solve for and and b:

8 = ab⁰ ⇒ 8 = a*1 ⇒ a = 818 = 8b ⇒ b = 18/8 ⇒ b = 2.25

Find k:

k = 8*[tex]2.25^{1.5}[/tex] = 27

Answer:

a=8

b=9/4

k = 27

Step-by-step explanation:

y = ab^x

We know that a is the initial value or when x = 0

We know the point (0,8) so a = 8

y = 8 b^x

Using the point (1,18) for x and y we can find b

18 = 8 * b^1

18/8 = b

9/4 = b

Now

y = 8 * ( 9/4) ^ x

Letting x = 1.5

k = 8 *(9/4)^1.5

k = 27

315,728 rouned to the nearest ten thousand

Answers

Answer:

320,000 is the answer

Step-by-step explanation:

have a good day!!

HELP pleaseee..!!!!!!!!!!!!!

Answers

Answer:

a) 9yd

b) 12ft

Step-by-step explanation:

a) find the square root

b) divide 48 by 4

help
dhgdfdfyfyfgfgfjfgfghfhgfdhgdfh

Answers

For the answers
1=26
2=17

Answer:

5=26

14=17

n=1

31n=31*1=31

- 4x < 68
A. Solve by adding 4 to each side.
B. The solution is x>-17.
a or b will mark as brainliest
(ignore the last thing)

Answers

Answer:

B.x>-17

Step-by-step explanation:

1. Divide both sides by -4:

-4x/-4<68/-4

2.< will change to > since you are dividing by negative

3.your final answer will be x>-17

On Saturday, the fruit and juice bar was selling 20 glasses of fruit punch an hour. By 7 p.m., they had sold 180 glasses. If their goal was to sell more than 240 glasses of fruit punch, find the number of hours, h, they must stay open past 7 p.m. to make their goal.

Answers

Answer: A)  h > 3

======================================================

Explanation:

h = number of hours past 7 pm

20h = the number of glasses sold after h hours

20h+180 = adding on the 180 glasses already sold

20h+180 > 240 is the inequality to set up

Let's solve for h

------------------

20h+180 > 240

20h > 240-180

20h > 60

h > 60/20

h > 3

They must remain open more than 3 hours past 7 pm in order to sell more than 240 glasses of juice.

The earliest that they can close is 11 pm (since 4+7 = 11).

--------------------

If h = 3, then

20h+180 = 20*3 + 180 = 240

meaning that they haven't reached their goal of getting over 240

So they have to stay open that extra hour to get past 240.

HELP HELP HELP HELP HELP

Answers

Answer:

C

Step-by-step explanation:

7x+ 11x=90

because the whole angle is

90 which makes it the answer Wich make the X a number we don't know what it is but know it helps make the exaction =90

Answer:

option c is correct

Step-by-step explanation:

7x + 11x =90 degree (being perpendicular)

18x=90

x=90/18

x=5

Which are correct representations of the inequality 6x2 3 + 4(2x – 1)? Select three options.
12 2x
6x > 3 + 8x - 4
O
-1.5
-1
-0.5
0
0.5
1
1.5
-1.5
-0.5
0
0.5
1.5
-1.5
-1
-0.5
0
0.5
1
1.5

Answers

8x provides distant of issue so 0 and -1.5 evaluate

The proper way to express the above inequality is 6x4(2x-1) is x2.

What is inequality?

A mathematical statement that has two mathematical expressions on each side of the inequality sign is called an inequality. The indicators for inequality are,, >, or.

6x4(2x-1) is the given inequality.

So, 6x≥8x-4

6x off of both sides gives us

6x-6x≥8x-4-6x

0≥8x-6x-4

0≥2x-4

4 added on both sides give us

0+4≥2x-4+4

4≥2x

By dividing both sides by half, we obtain

(1/2)×4≥(1/2)×2x

2≥x

x≤2

Therefore, x≤2 is the correct representation.

Learn more about inequality here:

brainly.com/question/19498063

#SPJ7

help ASAP ! it’s due soon

Answers

option

(c) is correct

Step-by-step explanation:

jsjsjd

your answer would be C

The number -2 is a solution to which of the following inequalities?
x + 7 > 5
-3 x < 1
x - 7 < -4

Answers

The answer is the third one, x - 7 < 4

To find it all you need to do is plug in -2 for x and see if the inequality is true.

When you do the math for the first one, you get 5 > 5. That is not true as 5 is equal to 5, not greater.

For the second one you get 6 < 1. Again, that is not true as 1 is not greater than 6.

Finally, you get -9 < -4 for the third one. This one is correct as -9 is a smaller number than -4.

Answer:

The answer is the third one, x - 7 < 4

Step-by-step explanation:

The temperature on a thermostat is set to 72∘F, but the actual temperature can vary by as much as 2∘F. Let x represent the actual temperature to create an absolute value inequality to determine the range of possible-actual temperatures.
Any help is much appreciated :)

Answers

This is your answer :)

Using an absolute value inequality, it is found that the range of temperatures is between 70ºF and 74ºF.

---------------

The absolute value function measures the distance of a point to the origin, and is defined by:

[tex]|x| = x, x \geq 0[/tex]

[tex]|x| = -x, x < 0[/tex]

---------------

The actual temperature is x, on a thermostat set to 72ºF. It can vary by as much as 2ºF, which means that the absolute value of the difference between x and 72 is at most 2, that is:

[tex]|x - 72| \leq 2[/tex]

Solving the inequality:

The term is both greater or equal to -2 and at most 2, thus:

[tex]x - 72 \geq >= -2[/tex]

[tex]x \geq 70[/tex]

And

[tex]x - 72 \leq 2[/tex]

[tex]x \leq 74[/tex]

Thus, the range of temperatures is between 70ºF and 74ºF.

A similar problem is given at https://brainly.com/question/24514895

Other Questions
The 21st century of public library service for today's generation 500 to 800 words in article features in ghana The sum of triple a number and 5 is 40. How did the United States' role as a world leader change after September 11, 2001 Some functions of cellular organelles are given below.a)Protein synthesisb)Cellular respirationc)Detoxify certain compoundsThe organelles that perform the above functions are given respectively in which option?(a) Mitochondrium,Ribosome, Plastid(b) Ribosome,Mitochondrium, EndoplasmicReticulumO (c) Ribosome, Golgy body,Endoplasmic reticulum(d) Ribosome, Plastid,Endoplasmic reticulum The hall is a rectangle that measures 4m long by 90 cm wide what is the area what is the triceratops animal kingdom A molecule of composition is replicated in a solution containing unlabeled (not radioactive) GTP, CTP, and TTP plus adenine nucleoside triphosphate with all its phosphorus atoms in the form of the radioactive isotope 32P. Will both daughter molecules be radioactive Given the reaction at equilibrium:H2(g) + Cl2(g) 2HCl(g) + heatThe equilibrium will shift to the right when there is anincrease inA temperatureB pressureC concentration of H2(g)O concentration of HCl(g) Given the triangle below, what is the measure of x to the nearest foot? What is the length of the hypotenuse of the right triangle in the diagram? Change fraction ( 11/4) from an improper fraction to a mixed number. An object with mass m = 0.56 kg is attached to a string of length r = 0.72 m and is rotating with an angular velocity = 1.155 rad/s. What is the centripetal force acting in the object? According to the passage how have humans primarily used artificial light In a group discussion, a participant says, I hate walking. Why walk when you can just get a ride? Write three to four sentences in which you share your own viewpoint. Be sure to provide evidence and refer to the passage you just read. SHOW WORK PLEASEMarion and her friends are driving to see a concert at the stadium, a trip of 145 miles. They drive for 2 hours at a steady rate of 40 miles per hour and then stop for gasoline. How much further do they have to travel to arrive at the stadium? What is the answer. Kinda freaking out Read the following passage:"It didn't matter how many times I wrapped the scarf over my face my nose hairs were going to stay frozen until I got on the bus. The winter wind swirled around the shelter and down my coat collar no matter which way I turned. I tried to remember why I'd thought moving to Wisconsin was a good idea. I could notWhat is the point of view in this reading?Third-personC First-personC Fourth-personC Second-person 4x-1divide by 2=x+7what is x Acme Air Compressor Company has decided to limit the number of compressors it will supply to some of its distributors that it suspects are selling a line of compressors made by one of its competitors. Acme is using its referent power in its distribution channel.a. Trueb. False find the sum of this infinite geometric series.HELP!!